क्या हम (सैद्धांतिक रूप से) ब्लैक होल को इतना मजबूत कर सकते हैं कि यह केन्द्रापसारक बल से अलग हो जाएगा?


26

मैं ब्लैक होल के जीवन में शामिल बलों की कल्पना नहीं कर सकता। तो कृपया, मुझे यह पता लगाने में मदद करें कि क्या इस विशिष्ट तरीके से ब्लैक होल को नष्ट करना संभव है या नहीं।


मुझे यकीन नहीं है, लेकिन मुझे लगता है कि इसके स्पिन को कुशलता से बढ़ाने के लिए, आपको इस पर सामान फेंकने की ज़रूरत है, और यदि आप करते हैं तो आप इसके द्रव्यमान को भी बढ़ाएंगे; जीआर की मजबूत क्षेत्र सीमा में, संबंधित द्रव्यमान वृद्धि केन्द्रापसारक बल को अलग करने से रोकती है। सबसे अच्छे रूप में यह 'ए' के ​​साथ केर ब्लैक होल बन जाता है, जो कि लगभग 1 पर है। ब्लैक होल के लिए, गुरुत्वाकर्षण हमेशा जीतता है! (यदि आप चीजों को फेंकने के बिना इसे टोकना चाहते हैं, तो आपको काफी दूर होना होगा और यह बहुत कुशल नहीं होगा)
क्रिस करें

जवाबों:


23

क्या हम (सैद्धांतिक रूप से) ब्लैक होल को इतना मजबूत कर सकते हैं कि वह केन्द्रापसारक बल से अलग हो जाएगा?

केर-न्यूमैन (घूर्णन, आवेशित, पृथक) द्रव्यमान का ब्लैक होल , कोणीय संवेग और आवेश , घटना क्षितिज का सतही क्षेत्र जहां । एक चरम ब्लैक होल तब होता है जब । इसके अलावा, अगर ब्लैक होल अधिक ओवरस्पून या ओवरचार्जेड है, तो एक "ओवरएक्सटर्मल" केर-न्यूमैन स्पेसटाइम है, जो वास्तव में ब्लैक होल नहीं होगा, बल्कि एक नग्न विलक्षणता होगी।जे क्यू = 8 एम [ एम 2 + ( एम 2 - एक 2 - क्यू 2 ) 1 / 2 - क्यू 2 / 2 ] , एक = जम्मू / एमएमजम्मूक्यू

=8एम[एम2+(एम2-2-क्यू2)1/2-क्यू2/2],
=जम्मू/एमएम2=2+क्यू2

इस प्रकार, मैं आपके प्रश्न की व्याख्या करते हुए पूछता हूं कि क्या ब्लैक होल को चरम सीमा तक और उससे आगे तक फैलाया जा सकता है, ताकि घटना क्षितिज को नष्ट किया जा सके। यह बहुत संभव है कि यह नहीं किया जा सकता है।

वाल्ड ने 1974 में साबित किया कि एक कोणीय गति को बढ़ाने की कोशिश करने के लिए एक ब्लैक होल में एक मक्खियों के रूप में यह एक अतिवादी ब्लैक होल के समीप होता है, इस प्रक्रिया को जारी रखना जितना कठिन है: एक तेजी से घूमता हुआ ब्लैक होल इसे पीछे हटा देगा। इसे चरम सीमा से परे ले जाएगा। अन्य योजनाएं हैं, और हालांकि मुझे शास्त्रीय सामान्य सापेक्षता के भीतर किसी भी पूरी तरह से सामान्य प्रमाण के बारे में पता नहीं है, इस तरह की योजनाओं की निरंतर विफलता ब्लैक होल डायनेमिक्स और थर्मोडायनामिक्स के बीच संबंध से अच्छी तरह से प्रेरित है।

उदाहरण के लिए, ब्लैक होल का हॉकिंग तापमान , जहां ब्लैक होल की सतह का गुरुत्वाकर्षण है। इस प्रकार, यहां तक ​​कि चरम सीमा तक पहुंचना थर्मोडायनामिक रूप से एक प्रणाली को पूर्ण शून्य तक ठंडा करने के बराबर है।κ = टीएच=κ/2π

κ=एम2-2-क्यू22एम(एम+एम2-2-क्यू2)-क्यू2

2

मेरे सिर के ऊपर से सारे गणित नहीं हैं, लेकिन मेरी वैचारिक समझ से यह संभव नहीं है।

ब्लैक होल्स का एक बड़ा पर्याप्त गुरुत्वाकर्षण आकर्षण होता है, यहां तक ​​कि प्रकाश भी "सतह" से परे अच्छी तरह से बच नहीं सकता है (यदि ब्लैक होल में कम पर्याप्त द्रव्यमान है कि यह अभी भी एक सतह है और एक विलक्षणता में ढह नहीं गया है)। इसका मतलब यह होगा कि इसे तेजी से स्पिन करना होगा कि सतह प्रकाश की गति से काफी तेजी से आगे बढ़ रही है ताकि बचने के लिए पर्याप्त रैखिक गति (अक्सर आम बोलचाल की भाषा में "केन्द्रापसारक बल" कहा जाता है), जिससे बचने के लिए सापेक्षता का सिद्धांत संभव नहीं है।

हॉकिंग विकिरण केवल इसलिए संभव है क्योंकि विद्युत चुम्बकीय विकिरण ब्लैक होल के "सतह" के लिए बहुत करीब से आगे बढ़ रहा है और प्रकाश केवल गुरुत्वाकर्षण द्वारा "तुला" हो सकता है, इसे एक स्टॉप तक नहीं खींचा जा सकता है।


2
इस उत्तर के साथ कई वैचारिक समस्याएं हैं। यह बताता है कि "सतह" घटना क्षितिज के अलावा कुछ और है - फिर यह क्या है? यह बताता है कि हॉकिंग विकिरण विद्युत चुम्बकीय जैसे बड़े पैमाने पर विकिरण तक सीमित है - सच नहीं है। यह सुझाव देता है कि ब्लैक होल प्रकाश को रोक नहीं सकते हैं - क्षितिज एक हल्की सतह है जो ऐसा करता है।
स्टेन लीउ


0

जहां तक ​​हमें पता है कि ऐसा कुछ भी नहीं है जो ब्लैक होल को रोक सके। इस धारणा को समझने के लिए आपको पहले ब्लैक होल के बारे में पता होना चाहिए । एक बार जब आप इसे समझ लेते हैं, तो आप देखेंगे कि कॉसमॉस की हमारी वर्तमान समझ के कारण कुछ भी नहीं है जो हम ब्लैक होल से कर सकते हैं।

यह सच है कि हॉकिंग विकिरण एक ब्लैक होल को प्रभावित कर सकता है, लेकिन यह केवल बहुत छोटे ब्लैक होल के लिए है।

वैसे, भौतिकी में कोई केन्द्रापसारक बल नहीं है - यह वास्तव में कई लोगों की गलत धारणा है। हालांकि, वहाँ केन्द्रक बल है

यहाँ छवि विवरण दर्ज करें



3
@ इल्मारियारोन नीचे-वोट के लिए धन्यवाद, हालांकि, आपको एहसास है कि कार्टून कुछ भी सही साबित नहीं होता है? Centripetal बल और केन्द्रापसारक के बीच एक अंतर है।
फंक्शन

2
निष्पक्ष होने के लिए, कार्टून बताता है कि केन्द्रापसारक बल जड़ता बल के रूप में है, जो सच है। संभवतः यह जड़त्वीय बलों को "वास्तविक नहीं" के रूप में व्याख्या करने के लिए समझदार है, लेकिन मुझे यकीन नहीं है कि इस प्रश्न पर क्या असर पड़ता है, क्योंकि गुरुत्वाकर्षण बल एक जड़त्वीय बल भी है।
स्टेन लीउ

यह "उत्तर" बहुत अस्पष्ट है, संदिग्ध बयान करता है, और मूल क्वेरी पर कोई वास्तविक असर नहीं है।
फ्लोरिन आंद्रेई

0

दिलचस्प। यह प्रक्रिया पहले स्थान पर ब्लैक होल के गठन को प्रभावित कर सकती है। एक घूर्णन तारे पर विचार करें जो मर जाता है और गुरुत्वाकर्षण बलों के कारण सिकुड़ने लगता है। जैसे-जैसे वह सिकुड़ती है, उसका सारा द्रव्यमान छोटे दायरे में अधिक से अधिक संकुचित होता जाएगा। इसके दो परिणाम होंगे: 1) गुरुत्वीय बल शरीर के विभिन्न भागों को घेरेगा, चौकोर त्रिज्या के व्युत्क्रम के साथ विकसित होगा और 2) इसकी घूर्णी गति कोणीय गति संरक्षण और विस्तार बल के कारण घूर्णन के कारण बढ़ेगी, त्रिज्या के विलोम के साथ बड़ा हो जाएगा। इसका मतलब है कि विस्तार बल एक अनुबंध की तुलना में अधिक तेज़ी से बढ़ेगा और कम से कम न्यूटनियन दृश्य में, विस्तार बल जीत जाएगा। इस दृष्टि से, ऐसा लगता है कि एक घूर्णन तारा कभी एक ब्लैक होल नहीं बनेगा ...


आपको यह लेख पसंद आ सकता है। slate.com/blogs/quora/2014/02/10/…
userLTK

-1

चलो निम्नलिखित प्रयास करें:

बलों के बराबर:

एफसी=मीटरv2आरएफजी=जीएममीटरआर2मीटरv2/आर=जीएममीटरआर2v2=जीएमआर

आररों=2जीएम/सी2

v2=जीएम2जीएम/सी2v=सी2/2v=सी2v=0,707सी

0,707सी

हालांकि, जब त्रिज्या का विस्तार होता है, तो कोणीय गति के संरक्षण से रोटेशन धीमा हो जाएगा ... इसलिए मुझे नहीं लगता कि यह अलग हो जाएगा ... शायद "ग्रे होल" बन जाए?

यदि कोई मौलिक गलती हुई हो तो कृपया मुझे क्षमा करें, मैं इस सब के लिए नया हूं ...: पी


3
यह न्यूटनियन दिखता है। क्या आपने ब्लैक होल के पास स्पेसटाइम की वक्रता पर विचार किया था? ब्लैक होल के लिए "केर समाधान" भी देखें।
गेराल्ड

मुझे "ग्रे होल" शब्द पसंद है।
userLTK

यह एक पुराना उत्तर है, लेकिन मुझे लगा कि मैं इसे यहां छोड़ दूंगा क्योंकि यह ब्लैक होल के कक्षीय वेग को संबोधित करता है। आपको बचने के लिए C पर घटना क्षितिज कताई में ब्लैक होल के सैद्धांतिक किनारे की आवश्यकता होगी, जो स्पष्ट रूप से असंभव है। physics.stackexchange.com/questions/207816/...
userLTK
हमारी साइट का प्रयोग करके, आप स्वीकार करते हैं कि आपने हमारी Cookie Policy और निजता नीति को पढ़ और समझा लिया है।
Licensed under cc by-sa 3.0 with attribution required.